Last visit was: 26 Apr 2024, 01:37 It is currently 26 Apr 2024, 01:37

Close
GMAT Club Daily Prep
Thank you for using the timer - this advanced tool can estimate your performance and suggest more practice questions. We have subscribed you to Daily Prep Questions via email.

Customized
for You

we will pick new questions that match your level based on your Timer History

Track
Your Progress

every week, we’ll send you an estimated GMAT score based on your performance

Practice
Pays

we will pick new questions that match your level based on your Timer History
Not interested in getting valuable practice questions and articles delivered to your email? No problem, unsubscribe here.
Close
Request Expert Reply
Confirm Cancel
SORT BY:
Date
Tags:
Show Tags
Hide Tags
VP
VP
Joined: 30 Jan 2016
Posts: 1232
Own Kudos [?]: 4560 [17]
Given Kudos: 128
Send PM
Senior Manager
Senior Manager
Joined: 15 Feb 2018
Posts: 424
Own Kudos [?]: 385 [0]
Given Kudos: 2380
Send PM
Intern
Intern
Joined: 07 Jan 2012
Posts: 48
Own Kudos [?]: 16 [0]
Given Kudos: 72
Location: Canada
Concentration: International Business, Entrepreneurship
GMAT Date: 04-30-2012
WE:Information Technology (Computer Software)
Send PM
Senior Manager
Senior Manager
Joined: 05 Aug 2017
Posts: 361
Own Kudos [?]: 447 [3]
Given Kudos: 277
Location: India
Concentration: Strategy, Marketing
WE:Engineering (Energy and Utilities)
Send PM
Re: A recent survey indicates that the average number of books read annual [#permalink]
2
Kudos
1
Bookmarks
KAPLAN OFFICIAL EXPLANATION

(B) Paradox (All EXCEPT)

Don’t predict the right answer to a Paradox EXCEPT question; instead, summarize the paradox in your own words before going to the answer choices. The discrepancy in this stimulus is signaled by the word “however,” so let’s get the gist of it: the average number of books read per capita has decreased, but bookstores have reported an increase in profits. Since this is a Paradox EXCEPT question, four of the answers will explain this discrepancy, and will therefore need to be eliminated. We don’t want to waste too much time trying to make predictions in this instance, so we should instead keep our discrepancy in mind as we head to the answer choices.

(A) If libraries haven’t been able to purchase as many popular contemporary novels, readers may have been forced to purchase them on their own at bookstores. So even though they may be reading fewer books on average, they’ll be purchasing those books themselves instead of checking them out from libraries. Eliminate.

(B) would explain why bookstores may have seen higher profits relative to other retail businesses, but that isn’t the paradox that needs resolution. Since (B) doesn’t explain how bookstores are making more money even though people seem to be reading less, this must be the odd man out—and therefore the answer we need to select.

(C) indicates that bookstores aren’t making more money by selling books, but by selling lots of coffee. This certainly resolves the discrepancy. Eliminate.

(D) Bookstores can sell fewer books and still earn more profits if they start selling books that are much more expensive than those sold previously. This explains how fewer books read per person can still lead to more money for bookstores. Eliminate.

(E) offers up yet another alternative source of income for bookstores—magazine sales. If bookstores are selling more magazines than they were previously, these stores can see increased profits even if people aren’t reading as many books. Eliminate.
e-GMAT Representative
Joined: 02 Nov 2011
Posts: 4348
Own Kudos [?]: 30797 [1]
Given Kudos: 635
GMAT Date: 08-19-2020
Send PM
Re: A recent survey indicates that the average number of books read annual [#permalink]
1
Kudos
Expert Reply
PinakiRDas wrote:
Not sure why it's not A - libraries still do purchase from book stores.


Yes, agreed. Still, One purchase to a library, especially that of a new novel, will cater to n number of readers. But if that many number of users purchase it due to the lack of availability of the same in the library, the bookstore profit increases.
Intern
Intern
Joined: 19 Dec 2022
Posts: 5
Own Kudos [?]: 2 [0]
Given Kudos: 30
Send PM
Re: A recent survey indicates that the average number of books read annual [#permalink]
Please explain option in E and the reason behind its invalidity
Target Test Prep Representative
Joined: 24 Nov 2014
Status:Chief Curriculum and Content Architect
Affiliations: Target Test Prep
Posts: 3480
Own Kudos [?]: 5137 [0]
Given Kudos: 1431
GMAT 1: 800 Q51 V51
Send PM
Re: A recent survey indicates that the average number of books read annual [#permalink]
Expert Reply
Onkar1998 wrote:
Please explain option in E and the reason behind its invalidity

This question is an EXCEPT question. So, the correct answer must be one that does not explain why bookstores' profits increased.

With that idea in mind, let's consider choice (E).

(E) Citing a lack of free time, many survey respondents indicated that they had canceled magazine subscriptions in favor of purchasing individual issues at bookstores when time permits.

Notice that the fact that people are purchasing individual magazines from bookstores rather than subscribing to magazines does explain why bookstore's profits increased. After all, when people subscribe to magazines, the get the magazines directly from the publishers, not from bookstores. So, by purchasing magazines individually rather than subscribing, people increase the revenue, and thus the profits, of bookstores.

Since this choice does explain why bookstore's profits increased, it cannot be the correct answer to this EXCEPT question.
ISB & IIM Moderator
Joined: 17 Mar 2021
Posts: 289
Own Kudos [?]: 121 [0]
Given Kudos: 123
Location: India
GMAT 1: 660 Q44 V36
GPA: 3.5
Send PM
Re: A recent survey indicates that the average number of books read annual [#permalink]
Hi Experts

GMATNinja KarishmaB MartyTargetTestPrep


Can anyone please explain me option B? According to me it should not be the answer

Reason is - The other factor for the increase in the profits of book store is decrease in shoplifting due to the machine
So it explains the paradox. How can it be the answer?
Tutor
Joined: 16 Oct 2010
Posts: 14823
Own Kudos [?]: 64926 [0]
Given Kudos: 426
Location: Pune, India
Send PM
Re: A recent survey indicates that the average number of books read annual [#permalink]
Expert Reply
Vatsal7794 wrote:
Hi Experts

GMATNinja KarishmaB MartyTargetTestPrep


Can anyone please explain me option B? According to me it should not be the answer

Reason is - The other factor for the increase in the profits of book store is decrease in shoplifting due to the machine
So it explains the paradox. How can it be the answer?



(B) Due to the installation of sophisticated new antitheft equipment, the recent increase in shoplifting that has hit most retail businesses has left bookstores largely unaffected.

RECENTLY, shoplifting has increased and has affected other stores. But since the bookstores installed "sophisticated new antitheft equipment," they have been left unaffected i.e. they did not get affected like the other stores.

The shoplifting phenomenon is a recent one and has not affected bookstores. That doesn't tell us why their profits have increased. In fact, if anything, the impact of this might be a decrease in profit in the short term at least since the sophisticated equipment would have been expensive and hence would have led to increase in cost. Over time, of course, it will more than pay for itself by not allowing theft.
The point is that it certainly doesn't explain the "increase in profit."
CEO
CEO
Joined: 07 Mar 2019
Posts: 2554
Own Kudos [?]: 1813 [0]
Given Kudos: 763
Location: India
WE:Sales (Energy and Utilities)
Send PM
Re: A recent survey indicates that the average number of books read annual [#permalink]
A recent survey indicates that the average number of books read annually per capita has declined in each of the last three years. However, it also found that most bookstores reported increased profits during the same period.

Each of the following, if true, helps to resolve the survey’s apparently paradoxical results EXCEPT:

(A) Recent cutbacks in government spending have forced public libraries to purchase fewer popular contemporary novels. - WRONG. First aspect touched which, though doesn't resolve the paradox, tells us about why books read increased on per capita basis.
(B) Due to the installation of sophisticated new antitheft equipment, the recent increase in shoplifting that has hit most retail businesses has left bookstores largely unaffected. - CORRECT. Neither aspect touched thus nothing resolved.
(C) Over the past few years many bookstores have capitalized on the lucrative coffee industry by installing coffee bars. - WRONG. Helps resolve 2nd aspect.
(D) Bookstore owners reported a general shift away from the sale of inexpensive paperback novels and toward the sale of lucrative hardback books. - WRONG. 2nd aspect touched.
(E) Citing a lack of free time, many survey respondents indicated that they had canceled magazine subscriptions in favor of purchasing individual issues at bookstores when time permits. - WRONG. A bit confusing as in how it resolves either of the aspects. But new customers purchased books so 2nd aspect is resolved.

Two aspects around which this passage revolves are:
1. Increased per capita books read
2. Increased profits of bookstores.
How come this is possible? The choice that doesn't touches either of the two aspects is most likely the correct answer.

Answer B.
GMAT Club Verbal Expert
Joined: 13 Aug 2009
Status: GMAT/GRE/LSAT tutors
Posts: 6921
Own Kudos [?]: 63671 [1]
Given Kudos: 1774
Location: United States (CO)
GMAT 1: 780 Q51 V46
GMAT 2: 800 Q51 V51
GRE 1: Q170 V170

GRE 2: Q170 V170
Send PM
Re: A recent survey indicates that the average number of books read annual [#permalink]
1
Kudos
Expert Reply
Vatsal7794 wrote:
Hi Experts

GMATNinja KarishmaB MartyTargetTestPrep

Can anyone please explain me option B? According to me it should not be the answer

Reason is - The other factor for the increase in the profits of book store is decrease in shoplifting due to the machine

So it explains the paradox. How can it be the answer?

(B) doesn't say that there's been a DECREASE in shoplifting. In fact, (B) very specifically states that the recent increase in shoplifting has left bookstores largely UNAFFECTED... and why unaffected? Because the bookstores had installed sophisticated new antitheft equipment.

Ignoring installation costs, (B) might explain why the bookstores didn't see a decrease in profits. But preventing a decrease is not the same as causing an increase.

And remember, we need to explain why profits have increased EVEN THOUGH the average number of books read annually per capita has declined. You might eliminate shoplifting at your bookstore with fancy equipment, but if nobody wants to read, you still won't make money.

Without more information or further assumptions, (B) isn't enough to explain the discrepancy. So it's our answer.

I hope that helps a bit!
Intern
Intern
Joined: 10 Oct 2023
Posts: 9
Own Kudos [?]: 1 [0]
Given Kudos: 67
GMAT Focus 1:
705 Q87 V83 DI85
Send PM
Re: A recent survey indicates that the average number of books read annual [#permalink]
Confused between option A & B. In a fewer books does not explain how profit increased for book stores and reading per capita decrease. Where as B explains the paradox that if theft decreased the profit may increase even if sales has reduced or not increased.
Tutor
Joined: 11 Aug 2023
Posts: 823
Own Kudos [?]: 1418 [1]
Given Kudos: 75
GMAT 1: 800 Q51 V51
Send PM
A recent survey indicates that the average number of books read annual [#permalink]
1
Kudos
Expert Reply
MrMBB wrote:
Confused between option A & B. In a fewer books does not explain how profit increased for book stores and reading per capita decrease. Where as B explains the paradox that if theft decreased the profit may increase even if sales has reduced or not increased.

In Critical Reasoning, we have to be super careful about details of what the passage and answer choices say.

In this case, (A) doesn't say just, "fewer books." Rather, it indicates that libraries specifically have purchased fewer books.

So, since libraries lend books and thus multiple people read one library book, the fact that libraries have purchased fewer books would explain how it could be that, even though the average number of books read annually per capita has declined, most bookstores reported increased profits. After all, if people are not able to borrow "contemporary novels" from libraries, if they want to read them, they have to purchase them from bookstores.

Then, in the case of (B), notice that the choice doesn't say, "theft decreased." Rather it says the recent increase in shoplifting has left bookstores "largely unaffected." That information means that theft from bookstores has not increased, in other words, has not changed.

The fact that theft has not changed would not explain the increased profits.

So, (B) is the correct answer.
GMAT Club Bot
A recent survey indicates that the average number of books read annual [#permalink]
Moderators:
GMAT Club Verbal Expert
6921 posts
GMAT Club Verbal Expert
238 posts
CR Forum Moderator
832 posts

Powered by phpBB © phpBB Group | Emoji artwork provided by EmojiOne